LSAT and Law School Admissions Forum

Get expert LSAT preparation and law school admissions advice from PowerScore Test Preparation.

 voodoochild
  • Posts: 185
  • Joined: Apr 25, 2012
|
#6276
GGTR,
It is moot to argue with you. If you feel that you have understood my question, please answer it. If not, please keep your suggestions to yourself. I would rather get beatings from experts than from a poster who has mysteriously "lurked" (really?) for "ages" and who posts demeaning or insulting comments, with no intention to help others.

My heart says that my question is valid. The assumption that's written in A) goes AGAINST the premise in statement#1 of the argument. It's a fact. No one can deny it. I asked this question to two other LSAT experts yesterday via email (Powerscore's competitors - their company name ends with the letter 's') - they were clueless, and I was asked to move on because they didn't have any answers. In fact, one of them said, I am correct. A) can be an assumption only if we ignore the first statement of the argument. I don't want to disclose their names, or their company name. I know that I am correct, but I am missing something.

Voodoo Child

[NB - Gender thing: sorry about that. You missed my point again - I wanted to give you respect. I know that there are better ways, but in between my office-related madness, I ended up saluting you with "Mr". --- Sorry about that.]
User avatar
 Dave Killoran
PowerScore Staff
  • PowerScore Staff
  • Posts: 5925
  • Joined: Mar 25, 2011
|
#6292
Hi Voodoo,

Let me add a few points here, which may help clarify some of the issues (or maybe not, who knows :-D ).

First, as you and I have talked about before, you are trying to turn this into a very clear black-and-white argument and answer set. A few previous times we've discussed that, and I think you know that there is an inherent danger in doing that because it means you fit the argument into your beliefs, instead of taking the argument on its own terms. How does that play a role here? One significant way is that you've taken the first line of the stimulus and made into a clear-cut cause and effect, where the loaning of less money caused the decline (and, I think, based on your comments, is the sole cause). But that's not what the premise says. It says that banks contributed to the decline. That's a big difference, and when you say "One way we can argue that because we already know the fact that less money =causes=> economic decline, why are we doubting it?" and "However, the premise clearly states that bank loaned less money :arrow: downturn.", you're missing their point--they never said it was this clear (and, if I've misunderstood your point, my apologies; your argument wasn't very clear to me because I felt you changed their meaning).

Second, I'm not sure you read Nikki's answer closely enough--he's dead on in his assessment of the argument. I get the feeling you have your view of this argument, and you want us to explain your view--but we can't do that :) Your view doesn't reflect what LSAC thinks, and that's all we care about (no offense to you, but they score the tests, so they are always right!).

Finally, let me give you my explanation of (A). Maybe that will help, maybe not, but keep in mind that my explanation reflects what the test makers undoubtedly think, and understanding them is our goal.
  • Premise: Prior to downturn, regulatory standards were tightened.

    Premise: During the downturn, banks loaned less money

    Assumption: Tightened regulatory standards caused the loaning of less money.

    Conclusion: Regulatory standards loosened will cause loaning of more money.
That assumption is key, and using causal arrows look like:

Tightened regulatory standards :arrow: caused the loaning of less money

(A) is a classic Defender answer, and one that eliminates an alternate cause for the loaning of less money. (A), when negated, says that the real cause of loaning less money wasn't the tightened standards, but the actual decline itself which decreased deposits, which then caused the banks to have less ability to loan money. In other words, if this is negated, the standards weren't the problem, the downturn was, in which case the conclusion about loosening standards would be undermined. This explanation, by the way, is an expansion of the exact points Nikki made.

Anyway, those are my thoughts. Thanks!
User avatar
 Dave Killoran
PowerScore Staff
  • PowerScore Staff
  • Posts: 5925
  • Joined: Mar 25, 2011
|
#6313
Hi Voodoo,

I wanted to close out this discussion by addressing this part of your last post:
  • "The assumption that's written in A) goes AGAINST the premise in statement#1 of the argument. It's a fact. No one can deny it."
Not to be contrary, but it doesn't go against it :-D Let's look at it slightly more closely.

It seems to me like you are conflating different ideas. (A) just says that the downturn didn't affect the deposits available to the bank.

The first premise said that banks loaned less money.

While these two ideas are related (deposit totals affect the amount a bank can loan), they are different concepts. Why the bank loaned less money is the question, and there are different possibilities (tightened regulations, etc). For example, let's say I have $1M on deposit as a basis for my loans, and regulations normally allow me to loan out 10X my deposits. Do I have to loan out $10M? No.

The argument says that banks loaned out less than $10M because tightened regulations made it impossible for them to do so (either by lowering the multiple or perhaps by tightening standards of who can get loans, which restricted the pool). But what if I didn't have $1M on deposit? That could cause me to loan less, and such a fact would hurt the causal explanation that it was the regulations. Answer choice (A) takes away that explanation, and is thus correct.

Ok, I think that should tie this on up on all fronts. Thanks!
 reop6780
  • Posts: 265
  • Joined: Jul 27, 2013
|
#16100
I got this one right by choosing answer A.

Since the stimuli establishes a causal relation between regulatory standard and banks' loaning, it assumes there is no other cause for banks' loaning.

Answer A eliminates a possible cause for banks' loaning.

My problem is due to a little difference in timing.

The stimuli states that regulatory standards were tightened PRIOR TO the downturn.

I was wondering why it would be even necessary to mention answer A because the downturn did not occur when banks's loaning decreased.

I guess "a significant decrease in the total amount of money" could happen before the tightened regulatory standard so that it would be worth removing possible cause ?
 Jon Denning
PowerScore Staff
  • PowerScore Staff
  • Posts: 907
  • Joined: Apr 11, 2011
|
#16165
Hey reop,

Yeah, this is simply a case where a causal argument's assumption is that an alternate cause doesn't exist. The argument is that the lending of money by banks is influenced by the regulatory standards, so that when the standards were tightened lending decreased, and if the standards are relaxed lending will go up. The timing of that is fine--standards tightened prior to lending decreasing in the downturn, and presumably relaxing them in the future would see the opposite effect. The timing of A is fine as well: lending decreased during/after the downturn (and after tightened regulations), so if A says the downturn didn't produce another possible cause of the decreased lending (less money on hand to lend), then it eliminates a possible alternate cause and is an assumption.

Thanks!
 Elsbeth123
  • Posts: 1
  • Joined: May 26, 2020
|
#75710
Hi! I’m hoping you can talk about the difference between answer choices A and D. I narrowed it down to these two as choices that illuminate a cause other than regulations as to why banks were lending less, but don’t really know how to articulate what’s going on in each and why A is a better defender. Thank you for your help!
 Frank Peter
PowerScore Staff
  • PowerScore Staff
  • Posts: 99
  • Joined: May 14, 2020
|
#75738
Hi Elsbeth,

With an assumption question like this one, prephrasing can be a very effective technique for identifying the issue at hand. We need to think about what piece of information this argument might be relying upon. What we get in this argument is a cause and effect relationship - the author is saying that there was an economic downturn, which was exacerbated by regulation, and relaxing the regulations will lead to the banks lending more money. That assumes that it was the regulation, and not the downturn, that was the sole cause for banks lending less money. So, when I read a question like this, my prephrase is often along the lines of that I'm looking for an answer choice that confirms that it was in fact the cause that the author has identified, and not an alternate cause, that led to the stated effect. To put it another way, we want an answer choice that confirms that it wasn't the downturn that led to the banks lending less money - it was the regulation.

Answer choice (A) does that. If we negated this answer choice (applying the Assumption Negation technique) so that it instead was saying that the downturn caused a significant decrease in the total amount of money on hand, now our argument has a problem, which means we have found the correct answer choice, because we have found an answer choice that addresses an issue that the argument relies upon.

Answer choice (D), on the other hand, doesn't quite address the cause and effect relationship as effectively. It seems to be saying that if there is a decrease in lending during an economic downturn, it is not significant. That doesn't really matter to us here - if we chose this answer choice, we are still leaving open the issue of whether it was the regulation or the downturn itself that led to a decrease in lending.

Get the most out of your LSAT Prep Plus subscription.

Analyze and track your performance with our Testing and Analytics Package.